Recherche sur le blog!

Affichage des articles dont le libellé est 1991. Afficher tous les articles
Affichage des articles dont le libellé est 1991. Afficher tous les articles

Concours Physique ENSAM (Options T et TA) 1991 (Énoncé)

Electricité ‑ Optique ‑ Mécanique
(Options T et TA) Durée : 4 h
ELECTRICITE
PREMIERE PARTIE
On considère le circuit de la figure E.1. dans lequel l'interrupteur Tr est fermé depuis un temps suffisamment long pour que le régime permanent soit établi. On s'intéresse au régime transitoire qui suit l'ouverture de l'interrupteur à l'instant t=0.
1.1 Etablir l'équation différentielle concernant vs en exprimant ses coefficients en fonction de L, C, r, R.
1.2 Déterminer numériquement ces coefficients à partir des valeurs numériques suivantes:
E = 15 volts, r = 5 ohms, L = 0,1 henry, C = 1000 microfarads, R = 200 ohms.
1.3 Résoudre cette équation différentielle et exprimer vs = f(t) sous la forme
${v_s} = A - B{\rm{.}}{e^{ - \alpha t}}{\rm{.}}\cos \left( {\omega t + j} \right)$
Représenter sommairement vs = f(t).

DEUXIEME PARTIE
Etude d'un variateur élévateur de tension continue.
On étudie le fonctionnement du dispositif de la figure E.2. destiné à délivrer aux bornes d'une résistance R une tension vs, dont les variations vs devront rester faibles, à partir d'une tension continue positive constante Ve. Le dispositif est construit autour d'un composant électronique Tr commandé par une tension périodique de période T.
On adopte les hypothèses suivantes:
‑ Tr est assimilable à un interrupteur parfait : de 0 à t1 l'interrupteur est fermé et la tension à ses bornes est nulle (u = 0) quel que soit le courant iT qui le traverse; de t1 à T l'interrupteur est ouvert, le courant qui le traverse est nul quelle que soit la tension u à ses bornes.
‑ D est une diode supposée parfaite: vD = 0 quand iD >0 (sens direct) et D = 0 quand vD <0 (sens inverse).
‑ L est une inductance supposée parfaite (résistance négligée).
‑ C est une capacité de forte valeur.
On s'intéresse uniquement au fonctionnement en régime périodique établi.
2.1. Quand l'interrupteur Tr est fermé, quel est l'état de la diode D ? (vs positive).
Que fait la diode D quand l'interrupteur Tr s'ouvre ? Justifier qualitativement votre réponse (préciser le rôle de l'inductance).
On admet que l'état de la diode D reste le même jusqu'à la fin de l'intervalle de temps pendant lequel Tr est ouvert.
2.2. Dans toute la suite du problème on néglige les variations du courant dans la résistance R, c'est-à-dire que ce courant est assimilé à sa valeur moyenne Is. Vs représente la valeur moyenne de la tension vs aux bornes de R et de C.
Justifier cette hypothèse par des relations et des considérations physiques simples concernant les éléments R et C; préciser les valeurs moyennes des courants ic et iD.
2.3. En étudiant successivement les deux états du circuit, montrer qu'en régime périodique établi, i varie entre deux valeurs extrêmes imin et imax. Donner deux expressions de i = imax ‑imin et, de leur égalité déduire le rapport Vs/Ve en fonction du rapport cyclique = t1/T.
2.4 Donner une représentation graphique sommaire de u, i, iD, iT, en fonction du temps, suivant le modèle de la figure E.3.
2.5 Application numérique: on donne Ve = 15 volts, on désire obtenir Vs = 48 volts; la fréquence de fonctionnement de l'interrupteur Tr est f = 20 kHz et L = 0,1 henry.
Préciser la valeur nécessaire de ainsi que de i.
2.6 Exprimer la valeur moyenne de i soit Im et la valeur moyenne de iD soit IDm ; quelle relation lie ces deux valeurs moyennes, exprimer cette relation à l'aide de = t1/T.
2.7 Exprimer la condition correspondant à l'hypothèse faite au 2.1 sur l'état de la diode D quant Tr est ouvert; montrer qu'on en déduit une limite inférieure Lmin de L.
Si R = 200 ohms, préciser numériquement Is, Im et Lmin avec les valeurs numériques déjà indiquées.
2.8 Donner une interprétation énergétique des phénomènes correspondant aux deux parties de la période T.
2.9 En négligeant toujours les variations du courant dans la résistance R, exprimer la quanti d'électricité Q échangée entre la capacité C et le reste du circuit pendant les deux parties de la période T; en déduire la variation vs de la tension vs aux bornes de C et de R.
Avec les valeurs numériques précédentes et C = 1000 microfarads, calculer vs.


OPTIQUE
Franges d'interférences à deux ondes
On propose le dispositif expérimental de la figure O.1. :
Deux miroirs plans M1 et M2, carrés de 4 centimètres de côté, ont un côté commun, leurs faces réfléchissantes sont en regard et leurs plans font entre eux un angle /2 ‑ avec = 2.10-3 radian.
Une source ponctuelle S, émettant une lumière monochromatique, de longueur d'onde = 6.10-7 mètre, est placée sur la droite d'intersection des deux plans de symétrie du dispositif et éclaire les faces réfléchissantes des deux miroirs.
Soit SA = d la distance de la source S au côté commun des deux miroirs;
on donne d = 10 centimètres.
1 ‑ Déterminer la région de l'espace où l'on peut observer des interférences entre :
‑ le faisceau réfléchi par M1, puis par M2
‑ le faisceau réfléchi par M2 , puis par M1
Pour quelle raison faut-il, ici, connaître la dimension des miroirs ?
2 ‑ On reçoit les deux faisceaux réfléchis sur un écran E perpendiculaire, en un point O, à AS et placé à une distance, AO = D, du côté commun des miroirs. On donne D = 1 mètre.
Justifier de l'observation de franges rectilignes sur E et préciser l'orientation de ces franges.
Calculer la largeur, sur l'écran, du système des franges observées, l'interfrange, le nombre de franges brillantes et le nombre de franges noires.

3 ‑ Montrer que l'on ne change pas la netteté des franges en remplaçant la source ponctuelle S par une fente fine parallèle au côté commun des deux miroirs. Montrer que cette netteté diminue si on élargit la fente.
La fente ayant une largeur de 8,25 10-5 mètre, représenter par un graphique les répartitions d'intensité données sur l'écran par le milieu et par chacun des deux bords de la fente. Montrer qu'en fait les franges ont disparu.
4 ‑ La fente étant, à nouveau, très fine, on place sur le trajet des faisceaux réfléchis, perpendiculairement à AS et à 15 cm du côté commun des miroirs, une lentille convergente L de distance focale f = 10 cm (figure O.2.).
Calculer la largeur totale et l'interfrange du nouveau système de franges obtenu sur l'écran E.
5 ‑ Pourquoi, à votre avis, ne vous a-t'on pas fait étudier des interférences qui peuvent être données plus directement par:
‑ le faisceau réfléchi par Ml
‑ le faisceau réfléchi par M2.
MECANIQUE
PREMIERE PARTIE
Deux masses m sont assujetties à se déplacer sur un axe horizontal, x'x, n'introduisant aucun frottement. Les deux masses sont d'une part, reliées entre elles par un ressort et d'autre part, reliées à deux points fixes A et B par deux autres ressorts (figure M.1). Les trois ressorts sont identiques, de masse négligeable et de même raideur k. La distance des points A et B est telle que la tension des trois ressorts est nulle lorsque les deux masses sont immobiles en leur position de repos. On désignera par x1 et x2 les déplacements de chacune des deux masses; x1 et x2 seront contrôlés algébriquement selon l'orientation de x'x précisée sur la figure. On posera o2 = k/m.
1.1 Etablir les équations différentielles qui lient les expressions instantanées de x1 et de x2.
1.2. Le système d'équations différentielles obtenu peut avoir pour solution des oscillations sinusoïdales de même pulsation pour x1 et pour x2 ; en exploitant ce fait, établir une équation donnant les seules pulsations possibles et calculer les valeurs de ces pulsations avec k = 25 N.m-1 et m = 5.10-2 kg.
1.3. Pour chaque pulsation précédente, quelle relation lie les expressions instantanées de x1 et x2 ? Préciser physiquement le mouvement des deux masses.
Comment lancer les deux masses à l'instant initial de leur mouvement pour obtenir chacune des oscillations sinusoïdales communes ?; Justifier physiquement de l'expression des pulsations obtenues.

DEUXIEME PARTIE
On considère maintenant le dispositif de la figure M.2.
Deux pendules simples identiques, de longueur l et de masse m, peuvent se mouvoir dans un même plan vertical autour de deux axes parallèles situés dans le même plan horizontal. Les masses m ont été réunies par un ressort de masse négligeable et de raideur k. La tension du ressort est nulle lorsque les pendules sont verticaux.
Dans tout le problème, on ne considérera que des mouvements de petite amplitude: le ressort reste horizontal et on peut alors confondre le déplacement des extrémités du ressort avec les composantes horizontales x1 et x2 des déplacements de chacune des masses mobiles par rapport à sa position d'équilibre. On contrôlera algébriquement x1 et x2 selon l'orientation de l'axe x'x précisée sur la figure. On notera g l'accélération de la pesanteur. On négligera tout phénomène de frottement.
2.1. Etablir, à nouveau, le système d'équations différentielles qui lient les expressions instantanées de x1 et x2.
2.2. Procéder à la même recherche que celle faite dans la première partie quant à l'existence de solutions sinusoïdales de même pulsation pour x1 et pour x2.
On posera: $\omega {{}_1^2} = \frac{{kl + mg}}{{ml}}$ et $\gamma = \frac{{kl}}{{kl + mg}}$
Exprimer les pulsations propres aux oscillations sinusoïdales communes de x1 et x2 en fonction de 1 et .
Calculer leurs valeurs avec l = 1 mètre et g = 9,80 mètre.seconde-2, m et k ayant les valeurs données à la première question.
2.3. Donner pour chaque pulsation précédente, les liaisons entre les expressions de x1 et x2 et commenter physiquement les mouvements.

Concours Physique Centrale-Supélec (M, P') 1991 Physique II (Corrigé)

Corrigé centrale 91 M-P'
Première partie.
I- Collision neutron-noyau
1/ Conservation de la qdm : $m{\vec V_1} = m{\vec V_2} + M{\vec w_2} \Rightarrow {\vec V_1} = {\vec V_2} + A{\vec w_2}$
Conservation de l'énergie: $\text{ }\!\!{\scriptscriptstyle 1\!/\!{ }_2}\!\!\text{ }m\vec{V}_{1}^{2}=\text{ }\!\!{\scriptscriptstyle 1\!/\!{ }_2}\!\!\text{ }m\vec{V}_{2}^{2}+\text{ }\!\!{\scriptscriptstyle 1\!/\!{ }_2}\!\!\text{ }M\vec{w}_{2}^{2}\Rightarrow \vec{V}_{1}^{2}=\vec{V}_{2}^{2}+A\vec{w}_{2}^{2}$
2/ De ${\vec V_1} = {\vec V_2} + A{\vec w_2}$, on tire : $\vec V_2^2 = {({\vec V_1} - A{\vec w_2})^2} = \vec V_1^2 + A\vec w_2^2 - 2A{V_1}{w_2}\cos \theta $
Soit $\cos \theta = \frac{{\vec V_1^2 - \vec V_2^2 + {A^2}\vec w_2^2}}{{2A{V_1}{w_2}}} = \frac{{A\vec w_2^2 + {A^2}\vec w_2^2}}{{2A{V_1}{w_2}}} = \frac{{{w_2}}}{{{V_1}}}\frac{{1 + A}}{2}$> 0 donc 0 < θ < π/2
En fonction des énergies : $\text{ }\!\!{\scriptscriptstyle 1\!/\!{ }_2}\!\!\text{ }m\vec{V}_{1}^{2}=\text{ }\!\!{\scriptscriptstyle 1\!/\!{ }_2}\!\!\text{ }m\vec{V}_{2}^{2}+\text{ }\!\!{\scriptscriptstyle 1\!/\!{ }_2}\!\!\text{ }M\vec{w}_{2}^{2}\Rightarrow {{E}_{1}}-{{E}_{2}}=\text{ }\!\!{\scriptscriptstyle 1\!/\!{ }_2}\!\!\text{ A}\,\text{m}\,\vec{w}_{2}^{2}$ et ${{E}_{1}}=\text{ }\!\!{\scriptscriptstyle 1\!/\!{ }_2}\!\!\text{ }\,\text{m}\,\vec{V}_{1}^{2}$
Alors $\cos \theta = \frac{{{w_2}}}{{{V_1}}}\frac{{1 + A}}{2} = \sqrt {\frac{{{E_1} - {E_2}}}{{A{E_1}}}} \frac{{1 + A}}{2}$donc $\frac{{{E_2}}}{{{E_1}}} = 1 - \frac{{4A{{\cos }^2}\theta }}{{{{(1 + A)}^2}}}$

II- Modèle des sphères dures.
1/ La force de contact passe par le centre d'inertie, donc la vitesse ${\vec w_2}$ sera dirigé suivant la réaction normale. On en déduit : $\sin \theta = \frac{b}{{{R_1} + {R_2}}}$
2/ Le paramètre d'impact peut varier entre 0 et la valeur R1 + R2. Ce qui correspond pour le centre du neutron à
à une cible de surface variant de 0 à (R1 + R2)2.
La probablité de recevoir un impact sur une couronne de rayon : b → b + db est :$\frac{{dP}}{1} = \frac{{2\pi bdb}}{{\pi {{({R_1} + {R_2})}^2}}}$
3/ Par définition: $ < - Ln\,[1 - K{\cos ^2}\theta ]{ > _b} = < - Ln\,[1 - \frac{{K{b^2}}}{{{{({R_1} + {R_2})}^2}}}]{ > _b} = - \int\limits_0^{{R_1} + {R_2}} {Ln[1 - \frac{{K{b^2}}}{{{{({R_1} + {R_2})}^2}}}]\;db} $
En posant $x = \frac{{K{b^2}}}{{{{({R_1} + {R_2})}^2}}}$⇒ $\frac{1}{K}\left[ {(1 - x)Ln(1 - x) - (1 - x)} \right]_0^K = \frac{1}{K}\left[ {(1 - K)Ln(1 - K) - (1 - K) + 1} \right]$
Ce qui donne : $1 + \frac{{1 - K}}{K}Ln(1 - K)$ cqfd . Il faut que 0 < K < 1 pour que la fonction aît un sens.
4/ On a obtenu $\frac{{{E_2}}}{{{E_1}}} = 1 - \frac{{4A{{\cos }^2}\theta }}{{{{(1 + A)}^2}}} = 1 - K{\cos ^2}\theta $ avec $K = \frac{{4A}}{{{{(1 + A)}^2}}}$< 1 si A > 1
on peut utiliser le résultat précédent : $K = \frac{{4A}}{{{{(1 + A)}^2}}} \Rightarrow 1 - K = {\left( {\frac{{A - 1}}{{A + 1}}} \right)^2}$
Donc coefficient de ralentissement : $\gamma = < - Ln\,[\frac{{{E_2}}}{{{E_1}}}]{ > _b} = 1 + {\left( {\frac{{1 - A}}{{\sqrt {2A} }}} \right)^2}Ln(\frac{{A - 1}}{{A + 1}}) = $
5/ a)La dérivée de γ vaut zéro pour : $0 = \left( {\frac{{1 - A}}{{\sqrt A }}} \right)\left\{ { - \left( {\frac{{{A^{1/2}} + {A^{ - 1/2}}}}{{2\sqrt 2 \;A}}} \right)Ln(\frac{{A - 1}}{{A + 1}}) - \left( {\frac{1}{{\sqrt A }}} \right)\left( {\frac{1}{{(A + 1)}}} \right)} \right\}$
Le terme entre crochet ne s'annulant pas, la racine est A = 1. On vérifiera que c'est bien un maximum pour le ralentissement.
b) A-N : 1H (A = 1) γ = 1 ; 2H (A = 2) γ = 0,725 ; 12C (A = 12) γ = 0,158 ; 238U (A = 238) γ = 0,008 ;
III- Application aux ralentissements des neutrons.
1/ Il y a ½ kT par degré de liberté, donc E300K = 3/2kT = 3,9.10−2 eV.
C'est très faible devant l'énergie initiale des neutrons. On peut considèrer les noyaux immobiles, sauf pour les dernières collisions.
2 a/ Avec $\gamma = < - Ln\,[\frac{{{E_2}}}{{{E_1}}}]{ > _b}$ et en écrivant : $\frac{{{E_n}}}{{{E_0}}} = \frac{{{E_n}}}{{{E_{n - 1}}}}\frac{{{E_{n - 1}}}}{{{E_{n - 2}}}}\; \cdots \frac{{{E_1}}}{{{E_0}}} \Rightarrow Ln\left( {\frac{{{E_n}}}{{{E_0}}}} \right) = \sum\limits_1^n {Ln\left( {\frac{{{E_p}}}{{{E_{p - 1}}}}} \right)} $
on a en raisonnant sur les valeurs moyennes : $Ln\left( {\frac{{{E_n}}}{{{E_0}}}} \right) = - n\gamma \Rightarrow {E_n} = {E_0}{e^{ - \gamma }}$
2b/ $n = - \frac{1}{\gamma }Ln\left( {\frac{{{E_{300K}}}}{{{E_0}}}} \right)$d'où 1H : n = 17 ; 2H : n = 24 ; 12C : n = 108 ; 238U : n = 214;
3a/ A une date t : $v(t) = \sqrt {\frac{{2E(t)}}{m}} $, la durée moyenne intercollision est: $\Delta t = \frac{\lambda }{{v(t)}}$et le nombre de collisions par unité de temps est : $\frac{{dn}}{{dt}} = \frac{1}{{\Delta t}} \Rightarrow \frac{{dn}}{{dt}} = \frac{1}{\lambda }\sqrt {\frac{{2E}}{m}} $.
3b/ L'équation $Ln\left( {\frac{{{E_n}}}{{{E_0}}}} \right) = - n\gamma $donne, en passant à la limite : $\gamma \,dn = - Ln\,[\frac{{E + dE}}{E}] = - \frac{{dE}}{E}$
soit : $\gamma \frac{{dt}}{\lambda }\sqrt {\frac{{2E}}{m}} = - \frac{{dE}}{E}$ ; en posant $\,y = \frac{E}{{\;{E_0}}}$ on a $\gamma \frac{{dt}}{\lambda }\sqrt {\frac{{2{E_0}}}{m}} = - \frac{{dy}}{{\;{y^{3/2}}}}$
3c/ L'intégration conduit à : $2\left[ {{y^{ - 1/2}} - 1} \right] = \frac{\gamma }{\lambda }t\,\sqrt {\frac{{2{E_0}}}{m}} $soit : $\,\sqrt {\frac{{{E_0}}}{E}} = 1 + \frac{\gamma }{{2\lambda }}t\,\sqrt {\frac{{2{E_0}}}{m}} $
4a/ On calcule d'abord $\,\sqrt {\frac{{{E_0}}}{E}} \approx 5000$ puis avec γ = 0,158 on trouve t = 120 µs .
On a toujours : $\,\sqrt {\frac{{{E_0}}}{E}} > > 1$ donc $\,t = \frac{{2\lambda }}{\gamma }\sqrt {\frac{m}{{2E}}} $ indépendant de E0.
4b/ La distance parcourue pendant dt est : $dx = v.dt = dt\sqrt {\frac{{2E}}{m}} $ et on a aussi $\gamma \frac{{dt}}{\lambda }\sqrt {\frac{{2E}}{m}} = - \frac{{dE}}{E}$
donc $dx = - \frac{\lambda }{\gamma }\frac{{dE}}{E} \Rightarrow x = \frac{\lambda }{\gamma }Ln\,{\frac{{{E_0}}}{E}_{300K}}$ on trouve ainsi x = 2,8 m.
On peut remarquer que cette distance corespond à nλ puisque $n = - \frac{1}{\gamma }Ln\left( {\frac{{{E_{300K}}}}{{{E_0}}}} \right)$.

Deuxième partie.
1a/ Avec $\xi \,\vec u = {A_1}M \to $ ⇒ le théorème d'Ampère donne$\vec B = \frac{{{\mu _0}I}}{{2\pi {\xi ^2}}}\vec k \wedge \xi \vec u$
1b/${A_1}M \to = $$(r - a\cos \theta ){\vec u_r} + a\sin \theta {\vec u_{^\theta }}$⇒$\vec B = \frac{{{B_0}}}{{{\xi ^2}}}\left\{ \begin{array}{l} - a\sin \theta \;{{\vec u}_r}\\(r - a\cos \theta \;){{\vec u}_\theta }\end{array} \right.$et${\xi ^2} = {a^2} + {r^2} - 2\,a\,r\cos \theta $
1c/ $\vec B' = {B_0}\left\{ \begin{array}{l} - \frac{{a\sin \theta }}{{{\xi ^2}}}\; = - \left[ {\sin \theta {\rm{ + 2}}u\sin \theta \;\cos \theta - {{\rm{u}}^{\rm{2}}}\sin \theta [1 - 4{{\cos }^2}\theta {\rm{]}}} \right]{\rm{ }}\\\frac{{(r - a\cos \theta \;)}}{{{\xi ^2}}} = \left[ {u - {\rm{cos}}\theta - 2{\rm{u}}\,{\rm{co}}{{\rm{s}}^{\rm{2}}}\theta + {{\rm{u}}^{\rm{2}}}\cos \theta [3 - 4{{\cos }^2}\theta ]} \right]\end{array} \right.$
2a/ Il faut faire une rotation de π et changer le signe du courant. Soit: $\vec{B}''(u,\theta )=-\vec{B}'(u,\theta +\pi )$
2b/ ${B_{1r}} = B{'_r}(u,\theta ) - B{'_r}(u,\theta + \pi ) = - 2{B_0}\left[ {\sin \theta - {{\rm{u}}^{\rm{2}}}\sin \theta [1 - 4{{\cos }^2}\theta {\rm{]}}} \right]$
${B_{1\theta }} = B{'_\theta }(u,\theta ) - B{'_\theta }(u,\theta + \pi ) = - 2{B_0}\left[ {{\rm{cos}}\theta - {{\rm{u}}^{\rm{2}}}\cos \theta [3 - 4{{\cos }^2}\theta ]} \right]$
en linéarisant : ${B_{1r}} = - 2{B_0}\left[ {\sin \theta + {{\rm{u}}^{\rm{2}}}\sin 3\theta } \right]$ et${B_{1\theta }} = - 2{B_0}\left[ {{\rm{cos}}\theta + {{\rm{u}}^{\rm{2}}}\cos 3\theta } \right]$
3a/ Il faut faire une rotation d'angle − 2π/3 et d'angle +2π/3 .
3b/ Donc ${B_r} = {B_{1r}}(u,\theta ) + {B_{1r}}(u,\theta - 2\pi /3) + {B_{1r}}(u,\theta + 2\pi /3)$
${B_\theta } = {B_{1\theta }}(u,\theta ) + {B_{1\theta }}(u,\theta - 2\pi /3) + {B_{1\theta }}(u,\theta + 2\pi /3)$
Or $\left\{ \begin{array}{l}\cos (\theta - 2\pi /3) + \cos (\theta + 2\pi /3) = - \cos \theta \\\sin (\theta - 2\pi /3) + \sin (\theta + 2\pi /3) = - \sin \theta \end{array} \right.$on a finalement:
${B_r} = - 2{B_0}\left[ {3{{\rm{u}}^{\rm{2}}}\sin 3\theta } \right]$
${B_\theta } = - 2{B_0}\left[ {3{{\rm{u}}^{\rm{2}}}\cos 3\theta } \right]$ donc $C = 6$
4a/ Ligne de champ: $d\vec \ell //\vec B \Rightarrow \frac{{dr}}{{rd\theta }} = \frac{{{B_r}}}{{{B_\theta }}}$ ⇒$\frac{{dr}}{r} = \frac{{\sin 3\theta }}{{\cos 3\theta }}d\theta \Rightarrow \,{r^3} = r_0^3/\cos 3\theta $
4b/ ci-contre : allure des lignes de champ.
4c/ Module $B(r) = 6{B_0}\;{r^2}/{a^2}$,
lignes isomodules B(r) = Cte sur un cercle de centre O
II- Action du champ sur un neutron
1a/ Pour un dipôle donc deux cas possibles : ${{E}_{//}}=-\,B$ et ${{E}_{\bot }}=\,B$
Soit en remplaçant B par $C{B_0}\;{r^2}/{a^2}$⇒ ${{E}_{//}}=-\text{ }\!\!{\scriptscriptstyle 1\!/\!{ }_2}\!\!\text{ }m{{\Omega }^{2}}{{r}^{2}}$ et ${{E}_{}}=\text{ }\!\!{\scriptscriptstyle 1\!/\!{ }_2}\!\!\text{ }m{{\Omega }^{2}}{{r}^{2}}$
1b/ La force est donnée par : $\vec F = \, - gr\vec ad\,{E_p}$ donc ${\vec F_{//}} = m{\Omega ^2}\,\vec r$ et ${\vec F_{\rlap{--} \rlap{--} \not /\rlap{--} /}} = - m{\Omega ^2}\,\vec r$
Pour confiner il faut une force de rappel, seuls les neutrons antiparallèles peuvent être confinés.
2a/ La RFD donne : ${\vec F_{\rlap{--} \rlap{--} \not /\rlap{--} /}} = - m{\Omega ^2}\,\vec r = m\frac{{{d^2}\vec r}}{{d{t^2}}} + m\frac{{{d^2}z}}{{d{t^2}}}\vec k$ ⇒$ - m{\Omega ^2}\,\vec r = m\frac{{{d^2}\vec r}}{{d{t^2}}}{\rm{ et }}\frac{{{d^2}z}}{{d{t^2}}} = 0$
2b/ L'intégration donne :$\,\vec r(t) = {\vec A_1}\cos \,\Omega t + {\vec A_2}\sin \Omega t$ où ${\vec A_1}{\rm{ et }}{\vec A_2}$ sont des constantes.
soit avec les conditions initiales: $z = {v_0}t$ et $\,\vec r(t) = {x_0}\vec i\cos \,\Omega t + \frac{{{u_0}}}{\Omega }\vec j\sin \Omega t$.
2c/ La trajectoire est une hélice d'axe Oz et de section elliptique.
3a/ Le neutron est confiné si le grand axe de l'ellipse est inférieur au rayon a; x0 étant plus petit que a il faut que:$a > \frac{{{u_0}}}{\Omega }$ soit encore :${u_C} = a\,\Omega $.
3b/ A-N: uC = 5,9 m.s−1ce qui donne EC = 18.10−8 eV et aussi TC = 1,4.10−3 K
Ce résultat justifie l'appellation neutron ultra-froids.
3c/ La fonction de répartition de Boltzmann permet de calculer la fraction de neutrons qui ont une énergie inférieure à la valeur calculée précédemment:
$F = \int\limits_0^{{E_C}} {\frac{1}{{\sqrt {2\pi } }}\frac{1}{{{{(kT)}^{3/2}}}}\sqrt E \exp ( - E/kT)\,dE} $
si T = 300 K << TC on peut simplifier ⇒$F \approx \int\limits_0^{{E_C}} {\frac{1}{{\sqrt {2\pi } }}\frac{1}{{{{(kT)}^{3/2}}}}\sqrt E \,dE} = \frac{1}{{\sqrt {2\pi } }}\frac{1}{{{{(kT)}^{3/2}}}}\frac{2}{3}\left[ {{E^{3/2}}} \right]_0^{{E_C}}$
Soit finalement : $F = \sqrt {\frac{3}{{4\pi }}} {\left[ {\frac{{{T_C}}}{T}} \right]^{3/2}} \approx {5.10^{ - 9}}$ donc extrémement faible.
4/ Les neutrons ont un mouvement de dérive suivant l'axe Oz. or les fils créant le champ magnétique ne peuvent être rééllement infinis. Le confinement n'a lieu que dans la partie centrale du dispositif et se termine lorsque les neutrons sortent du dispositif.
III- Amélioration du confinement

1a/ Pour les neutrons confinés : ${\vec F_{//}} = - m{\Omega ^2}\,\vec r$ avec maintenant $\vec r = $$O'M \to $$ = (\rho - R){\vec u_\rho } + z\vec k$
1b/ En cylindriques : $\vec a = (\ddot \rho - \rho {\dot \theta ^2}){\vec u_\rho } + (2\dot \rho \dot \theta + \rho \ddot \theta ){\vec u_\theta } + \ddot z\vec k$
1c/ Equations du mouvement : $\left\{ \begin{array}{l}\ddot \rho - \rho {{\dot \theta }^2} = - {\Omega ^2}(\rho - R)\\2\dot \rho \dot \theta + \rho \ddot \theta = 0\\\ddot z = - {\Omega ^2}z\end{array} \right.$
2a/ Compte tenu des conditions initiales: $\ddot z = - {\Omega ^2}z \Rightarrow z = {z_0}\cos (\Omega t) + \frac{{{V_0}}}{\Omega }\sin \Omega t$.
2b/ $2\dot \rho \dot \theta + \rho \ddot \theta = \frac{1}{\rho }\frac{{d({\rho ^2}\dot \theta )}}{{dt}} = 0 \Rightarrow {\rho ^2}\dot \theta = Cte = \rho _0^2{\omega _0}$ "mouvement projeté sur x0y à force centrale".
2c/ Il reste l'équation en ρ(t): $\ddot \rho - \rho {\dot \theta ^2} = \ddot \rho - \left( {\frac{{\rho _0^4\omega _0^2}}{{{\rho ^3}}}} \right) = - {\Omega ^2}(\rho - R)$
3a/ si ω0 = 0 alors θ = θ0 est constant : $\ddot \rho = - {\Omega ^2}(\rho - R) \Rightarrow (\rho - R) = ({\rho _0} - R)\cos \Omega t$,
c'est l'équation paramètrique (z(t),ρ(t)) d'une ellipse de centre O'.
3b/ si $\dot \theta = Cte = {\omega _0}$ alors ${\rho ^2} = \rho _0^2$, la trajectoire est sinusoïde dessinée sur un cylindre d'axe Oz.
La trajectoire sera fermée si la durée d'un tour est un multiple de la période, soit $\Omega = n{\omega _0}$.
4a/ Si $\rho = {\rho _m}[1 + \varepsilon (t)]$ alors l'équation en ε est :${\rho _m}\ddot \varepsilon - \left( {\frac{{\rho _0^4\omega _0^2}}{{\rho _m^3}}} \right)[1 - 3\varepsilon ] = - {\Omega ^2}({\rho _m} - R + {\rho _m}\varepsilon )$
4b/ La valeur moyenne correspond à ε = 0 : $ - \left( {\frac{{\rho _0^4\omega _0^2}}{{\rho _m^3}}} \right) = - {\Omega ^2}({\rho _m} - R)$on a
4c/ Par différence : ${\rho _m}\ddot \varepsilon + 3\left( {\frac{{\rho _0^4\omega _0^2}}{{\rho _m^3}}} \right)\varepsilon + {\Omega ^2}{\rho _m}\varepsilon = 0$ soit : $\ddot \varepsilon + 3\left( {\frac{{\rho _0^4\omega _0^2}}{{\rho _m^4}}} \right)\varepsilon + {\Omega ^2}\varepsilon = 0$
ce qui s'intègre en $\varepsilon (t) = {\varepsilon _0}\cos (\Omega 't + {\varphi _0})$ en posant : $\Omega ' = \sqrt {3\left( {\frac{{\rho _0^4\omega _0^2}}{{\rho _m^4}}} \right) + {\Omega ^2}} $.
Ce qui donne alors la vitesse angulaire: $\dot \theta = \frac{{\rho _0^2{\omega _0}}}{{{\rho ^2}}} \approx \frac{{\rho _0^2{\omega _0}}}{{\rho _m^2}}[1 - 2\varepsilon ]$.
4d/ Les trajectoires sont alors ses oscillations autour des sinusoïdes tracées sur un cylindre. La vitesse angulaire étant elle même oscillante.
5/ La pesanteur entaîne un mouvement de chute selon l'équation z = ½ gt2 qui s'ajoute aux oscillations. Au bout d'une période la "chute" vaut donc : $h = 2g{\pi ^2}/{\Omega ^2}$ .
On calcule alors : h = 5,6 mm, ce qui n'est pas négligeable.
___________________________

Concours Physique ENSAM (Option T) Thermodynamique-Chimie 1991 (Énoncé)

THERMODYNAMIQUE ‑ CHIMIE
Option T
( Durée 4 heures )
L épreuve comprend une partie Thermodynamique et une partie Chimie que les candldats devront obligatoirement traiter sur des copies séparées convenablement repérées.
THERMODYNAMIQUE
Cette partie de l'épreuve comprend deux exercices indépendants à traiter dans un ordre laissé au choix du candidat.
I
Un ensemble moteur destiné à un véhicule automobile est représenté schématiquement Figure 1. On admet que le fluide qui circule dans l'installation est de l'air assimilable à un gaz parfait dont les caractéristiques thermiques sont les suivantes:
‑ Capacité thermique massique à pression constante: cp = 1 kJ.kg-1.K-1
‑ Rapport des capacités thermiques à pression constante et à volume constant: γ = 1,4.
Le débit masse qm de l'air est égal à 0,9 kg.s-1.
L'installation comporte les éléments décrits ci‑dessous.
a) Un turbocompresseur TC de caractéristiques suivantes:
‑ Rendement mécanique: ηm = 0,95
‑ Température d'aspiration de l'air : t1 = 10°C
‑ Pression d'aspiration de l'air: p1 = 1 bar
‑ Rapport de compression : ( p2 / p1 ) = 4
‑ Compression de p1 à p2 : adiabatique
‑ Rendement indiqué de la compression par rapport à l'isentropique: ηsc = 0,9
${\eta _{SC}} = \frac{{{W_{i12'}}}}{{{W_{i12}}}}$
Wi12 : travail indiqué de la compression réelle.
Wi12' : travail indiqué d'une compression isentropique fictive entre l'état 1 et la pression P2 .
L'indice 2' désigne l'état final atteint.
b) Une turbine TU de caractéristiques suivantes:
‑ Rendement mécanique: ηm = 0,95
‑ Température d'admission de l'air: t4 =927°C
‑ Détente de p4 à p5 : adiabatique
‑ Rendement indiqué de la détente par rapport à l'isentropique: ηST = 0,81
${\eta _{ST}} = \frac{{{W_{i45}}}}{{{W_{i45'}}}}$
Wi45 : travail indiqué de la détente réelle.
Wi45' : travail indiqué d'une détente isentropique fictive entre l'état 4 et la pression p5.
L'indice 5' désigne l'état final atteint.
La turbine entraîne le turbocompresseur et la transmission du véhicule.
c) Un échangeur adiabatique E d'efficacité ε égale à 0,74.
L'efficacité est définic par le rapport:
$\varepsilon = \frac{{{t_3} - {t_2}}}{{{t_5} - {t_2}}}$

d) Une chambre de combustion CH de caractéristiques suivantes:
‑ Parois : adiabatiques
‑ Combustion : isobare
‑ Rendement de combustion: η C = 0,97
C = (Quantité de chaleur reçue par le fluide) / (Quantité de chaleur fournie par le combustible)
On néglige:
‑ les pertes de charge, d où p2 = p3= p4 et p5 = p6= pl
‑ les variations d'énergie cinétique et d'énergie potentielle,
‑ les variations de température dans les canalisations reliant les divers éléments,
‑ les variations de débit dues au combustible injecté.
1‑ Calculer la température t2 du gaz à la sortie du turbocompresseur ainsi que la puissance PC fournie à l'arbre du compresseur.
2‑ Calculer la température t5 du gaz à la sortie de la turbine et la puissance PT disponible sur l'arbre de la turbine. En déduire la puissance utile Pu reçue par la transmission du véhicule.
3‑ Calculer la température t3 du gaz à l'entrée de la chambre de combustion, le rendement global ηt de l'installation et le débit masse horaire qh du combustible dont le pouvoir calorifique est égal à 4.104 kJ.kg-1.
4‑ Calculer la température t6 à la sortie de l'échangeur E.
5‑ Calculer les entropies massiques s en kJ.kg-1.K-1 pour les états 1 - 2 - 3 -4 - S et 6 du fluide en prenant s = 0 pour l'état 1. Représenter le cycle d'évolution du fluide dans le diagramme entropique, en choisissant des échelles convenables sur les deux axes.
II
Une unité de dessalement de l'eau de mer destinée à l'alimentation en eau potable des membres de l'expédition française en Terre Adélie est schématiquement représentée Figure 2.
Son fonctionnement en régime permanent peut être décrit comme suit.
L'eau de mer entre en A dans un récupérateur RC où elle est réchauffée par la saumure chaude extraite en H de l'évaporateur E2.
Elle traverse ensuite successivement les condenseurs C2 et C1. Elle entre en D dans l'échangeur principal EP où elle reçoit de la chaleur foumie par une source exteme constituée par l'eau de refroidissement des moteurs Diesel de la centrale électrique de la base.
L'eau de mer ainsi préchauffée est introduite au point E dans l'évaporateur E1 où elle est soumise au vide correspondant à la température d'extraction, soit 50°C. Une évaporation partielle a lieu et la vapeur produite se condense dans le condenseur C1, produisant ainsi de l'eau distillée qui est extraite en continu en K
La saumure restante entre en F dans l'évaporateur E2 où règne un vide correspondant à la température d'extraction de 40°C. Une nouvelle évaporation partielle a lieu et la vapeur se condense dans le condenseur C2, produisant de nouveau de l'eau distillée qui est également extraite en continu en L.
Les températures suivantes sont données aux points correspondants de la Figure 2:
tA = 2°C ; tB = 26,5°C ; tE = 60°C ; tF = tG = 50°C ; tH = tI = 40°C
‑ Enthalpies massiques de la vapeur d'eau saturée:
à 50°C h = 2591 kJ.kg-l ; à 40°C h = 2573 kJ.kg-l
Ces valeurs correspondent à h = 0 pour le liquide saturant à 0°C.
‑ Capacité thermique massique de l'eau liquide douce ou salée: c = 4,186 kJ.kg-l.K-l.
Les parois de tous les éléments de l'installation sont supposées adiabatiques.

1‑ Calculer, pour 1 kg d'eau de mer entrant en A, les masses ml et m2 d'eau distillée extraites en régime permanent des condenseurs C1 et C2 aux points K et L.
2‑ Calculer les températures tJ, tC et tD aux points correspondants de l'installation.
3‑ L'unité produit 2800 kg d'eau distillée par jour. Calculer la puissance thermique Pth foumie par l'échangeur principal.
CHIMIE
Les parties A et B sont indépendantes. Elles seront traitées dans un ordre laissé au choix du candidat .
A ‑ On étudie l'équilibre homogène en phase gazeuse décrit par le schéma réactionnel ( 1 ):
CO + H2O CO2 + H2 (1 )
1- Etudier la variance du système et exprimer la constante d'équilibre Kp en fonction des pressions partielles des différentes espèces gazeuses.
2‑ Calculer Kp aux températures de 750K et 1500K pour l'équilibre (1).
Données :
‑ Enthalpies libres réactionnelles standard des équilibres homogènes en phase gazeuse décrits par les schémas réactionnels (2) et (3).
2CO + O2 $\rightleftarrows $ 2 CO2 (2)
2H2 + O2 $\rightleftarrows $ 2 H2O (3)
ΔG° (2) = ‑ 565260 + 173,5 T
ΔG° (3) = ‑ 493570 + 112 T
où les enthalpies libres sont exprimées en joules et les températures en kelvin. La pression de référence pour les espèces gazeuses est égale à 1 bar.
‑ Constante molaire des gaz parfaits: R = 8,314 J.mol-1.K-1.
3‑ Afin d'étudier l'évolution des systèmes gazeux constitués par des mélanges quelconques de dioxyde de carbone, de monoxyde de carbone, d'hydrogène et de vapeur d'eau, on représente leur composition en utilisant le diagramme carré décrit ci-dessous ( Figure 3.) Pour un mélange constitué de:
$n_{CO}$moles de CO; n$_{C{O_2}}$ moles de CO2; n$_{{H_2}}$ moles de H2; n$_{{H_2}O}$ moles de H2O
on définit les variables:
$x = \frac{{{n_{CO}}}}{{{n_{CO}} + {n_{C{O_2}}}}}$ et $y = \frac{{{n_{{H_2}}}}}{{{n_{{H_2}}} + {n_{{H_2}O}}}}$
x et y sont les coordonnées du point représentatif de la composition du mélange étudié.
3.1 Il est facile de vérifier que le point O (0,0) représente un mélange CO2 + H2O en proportions quelconques. Que représentent :
a) le point A (0,1)?
b) le point B (1,0)?
c) le point C (1,1)?
d) un point appartenant au segment OA?
e) un point appartenant au segment OB?
f) un point appartenant au segment AC?
g) un point appartenant au segment BC?
3.2 Exprimer Kp en fonction des valeurs de x et y à l'équilibre.
3.3 Représenter graphiquement sur le diagramme carré, le lieu des points correspondant aux divers mélanges à l'équilibre à T = 750K et à T = 1500K. Les courbes seront tracées point par point en faisant varier x à partir de zéro par incréments de 0,1.
4‑ On part d'un mélange initial à la température T K contenant (nCO)0 moles de monoxyde de carbone et (n$_{{H_2}O}$)0 moles d'eau.
4.1 Montrer que l'évolution du système vers son état d'équilibre à la température T K est représentée, sur le diagramme carré, par une droite dont on donnera l'équation.
4.2 Application : On part à 1500K d'un mélange contenant 2 moles de CO et 1 mole de H2O.
Représenter l'évolution du système vers l'équilibre et donner sa composition une fois cet équilibre atteint.
5‑ A 750K, un mélange à l'équilibre est tel que le rapport du nombre de moles d'hydrogène au nombre de moles d'eau est égal à 2. Donner le lieu des points représentant les mélanges initiaux ne contenant pas de dioxyde de carbone qui conduisent à cet état d'équilibre.

B‑ On considère, à la température de 298K, le système hétérogène formé par une atmosphère contenant du CO2 gazeux en contact avec une solution aqueuse contenant du CO2 dissous et des
ions hydrogénocarbonate HC03-. Les équilibres mis en jeu sont décrits de manière simplifiée par les schémas réactionnels (4) et (5) . On néglige l'équilibre faisant intervenir les ions carbonate.
CO2 gazeux $\rightleftarrows $ CO2 dissous (4)
CO2 dissous + 2 H2O $\rightleftarrows $ HC03- + H3O+ (5)
Pour l'équilibre (4), la concentration volumique molaire de CO2 dissous est reliée à la pression partielle p$_{C{O_2}}$ de CO2 gazeux par la relation suivante valable à la température de 298K:
[CO2] dissous = 0.035.p$_{C{O_2}}$ où p$_{C{O_2}}$ est exprimée en bars et [CO2] en mol.l-1.
La constante d'équilibre relative aux concentrations volumiques molaires de la réaction (S) sera notée Kc
1. Exprimer Kc en fonction de la pression partielle p$_{C{O_2}}$ et des concentrations volumiques molaires
[CO2] dissous, [HCO3-] et [H3O+].
2. Afin de déterminer la valeur de Kc à la température de 298K, on construit la cellule de mesure représentée Figure 4.
La demi-cellule A contient une solution aqueuse de HCl de concentration volumique molaire 0,1 mol.l-1. La demi-cellule B contient une solution aqueuse de NaHCO3 de concentration volumique molaire 0,001 mol.l-1. La solution est en équilibre avec une atmosphère gazeuse dans laquelle la pression partielle de CO2 est égale à 0,01 bar. Les deux demi-cellules sont reliées par un pont d'électrolyte pour lequel les différences de potentiel de jonction sont négligeables.
On mesure la différence de potentiel E entre deux électrodes à hydrogène immergées dans les deux solutions.
2.1 Exprimer E en fonction des valeurs pHA et pHB du pH des deux solutions.
Données: R = 8,314 J.mol-1.K-1
1 Faraday = 96487 C.
2.2 A la température de 298K, on mesure E = 350mV. En déduire la valeur de la constante d'équilibre Kc.

Concours Physique École de l’Air 1991 (Énoncé)

Ecole de l'Air 1991

Première épreuve de sciences physiques

Partie I

Un fil recti1igne infini f de dimensions transversales nég1igeables, placé dans 1e vide, porte des charges é1ectriques réparties uniformément, avec une densité 1inéique λ. Ce fil est para11èle à l'axe de coordonnées Oz et i1 a pour trace sur 1e p1an xOy le point F de coordonnes x = a et y = 0.
Pour 1es app1ications numériques de cette partie, on prendra :
$\lambda = {5.10^{ - 10}}{\rm{ }}C/m$ et a = 4 cm
On rappelle enfin que : ${\varepsilon _o} = \frac{1}{{36\pi {{.10}^9}}}$
1) En quelles unités est exprimée habituellement 1a permittivité ε ? Justifier 1e choix de ces unités.
2) Montrer que le champ é1ectrostatique E produit par ce fil est indépendant de z.
3) Déterminer 1e champ électrostatique en un point quelconque M du plan xOy en introduisant la variable FM = r.
On fera un schéma clair. App1ication numérique r = 4 cm.
4) V désignant 1e potentiel é1ectrostatique en M et V° le potentiel en 0, calculer 1a différence de potentie1 V-V°.
Faire l'application numérique.

Partie II

On étudie maintenant toujours dans le vide, le système constitué par 1e fil f associé à un second fil f' symétrique de f par rapport à Oz et portant des charges électriques uniformément réparties, avec la densité linéique -λ. La trace de f' dur le plan xOy est le point F'.
Pour les applications numériques de 1a partie II, on prendra encore $\lambda = {5.10^{ - 10}}{\rm{ }}C/m$ et a = 4 cm
l) M étant un point quelconque du p1an xOy, r sa distance à F et r' sa distance à F', V le potentiel en M et V° en 0, calculer dans ce nouveau cas la différence de potentiel V-V°. Déterminer la valeur de V° telle que V tende vers zéro quant M s'éloigne indéfiniment dans 1e plan xOy. Tracer à l'échelle 1es équipotentielles pour V = ± 14,5 V
2) Si l'on fait tendre a vers zéro, mais en gardant le produit (2λa) constant et éga1 à p, on obtient un "dipôle cylindrique" de moment p (p para11èle à Ox). Ca1cuIer 1e potentiel V(ρ,θ) en M. En déduire Eρ et Eθ du champ E en M.
On posera $\rho = OM$

Partie III

On se donne maintenant, toujours dans 1e vide, deux cercles Γ et Γ' du plan xOy, dont les centres respectifs C et C' sont situés sur Ox, symétriquement par rapport à 0. On désigne par R leur rayon, et on pose $OC = OC' = \ell {\rm{ avec }}\ell > R$
Pour les applications numériques de cette partie. on prendra $\ell = 5{\rm{ cm}}$ et R = 3 cm
l) Montrer que l'on peut placer les fils f et f' étudiés dans la deuxième partie de façon que Γ et Γ' soient des lignes équipotentielles et calculer en fonction de et de R la valeur de a et celle du rapport $\alpha = r'/r$ correspondant au cercle Γ.
Application numérique : calculer les valeurs de a et de α.

2) Si l'on impose le potentiel V1 de Γ. Calculer la densité linéique λ que l'on doit attribuer au fil f .
App1ication numérique : calculer λ pour V1 = 10 V.
3) On imagine maintenant que 1'on remplace les fils f et f' par deux cylindres conducteurs ayant pour sections droites Γ et Γ' et respectivement portés aux potentiels +V1 et -V1 . Qu'appelle-t-on équilibre électrostatique ? Montrer que 1a distribution de potentiel à l'extérieur de Γ et Γ' satisfait toutes les conditions de l'équilibre électrostatique.


Partie IV

On imagine maintenant N fils rectilignes et chargés comme précédemment (+ λ) répartis régulièrement sur un cylindre de rayon a, centré en 0. Calculer le potentiel en un point M à grande distance des fils. On fera un développement limité au second ordre en a/r. Cas où N = 8 .

Partie V

Cette fois on considère deux cylindres conducteurs très allongés identiques, de longueur L et de rayon R placés parallèlement à une distance a. Ces cylindres constituent une ligne bifilaire : on supposera que R << a << L et on ne tiendra pas compte des effets de bords.
l) Le cylindre (1) portant la charge électrique +Q et 1e cylindre (2) portant la charge -Q , calculer en tenant compte des approximations, les potentiels V1 et V2 des deux cylindres; on supposera que les charges sont uniformément réparties sur chaque cylindre et, on utilisera une valeur moyenne de la densité superficielle. On donnera au préalable et sans aucun calcul, une idée de la répartition des charges sur les cylindres.
2) Déterminer la quantité (capacité 1inéique) définie par : $Q/L = \gamma ({V_1} - {V_2})$
Calculer γ pour a = 4 cm et R = 2 mm.
3) Les deux cylindres étant initialement déchargés, sont portés aux potentiels V1 et V2 quelconques. Calculer les charges totales Q1 et Q2 que prennent les deux cylindres, et déterminer 1es expressions des quatre coefficients Cij définis comme suit: ${Q_i} = \sum {{C_{ij}}{V_j}} $
Cette écriture signifie que les charges Q1 et Q2 se mettent sous la forme : $\left\{ \begin{array}{l}{Q_1} = {C_{11}}{V_1} + {C_{12}}{V_2}\\{Q_2} = {C_{21}}{V_1} + {C_{22}}{V_2}\end{array} \right.$
6) Lorsque la 1ongueur L augmente indéfiniment, déterminer la 1imite des coefficients 1inéiques Cij/L .
Comparer ces valeurs à 1a capacité γ précédente.

Concours Physique Centrale M, P’ Physique II 1991 (Énoncé)

Concours d'admission 1991

M-P'

PHYSIQUE II
(4 pages dactylographiées)
ETUDE D'UN PIEGE A NEUTRONS
Le neutron est une particule sans charge électrique, il n'est donc pas possible de le piéger dans les anneaux de stockage traditionnels. Nous allons voir qu'il est néanmoins possible de confiner des neutrons très lents, appelés "ultra-froids", dans un champ magnétique approprié. La première partie du problème étudie le ralentissement des neutrons. La seconde partie s'intéresse au confinement des neutrons.
Les deux parties sont très largement indépendantes.
Données : Masse du neutron m = 1,67. 10−27 kg
Moment magnétique du neutron M = 9,66. 10−27 Am2
Constante de Boltzmann k = 1,38.10−23 .J.K−1
Electron-volt 1 eV = 1,6.10−19 J
Le référentiel du laboratoire sera supposé galiléen dans tout le problème.
<F(x)>x représente la valeur moyenne de la fonction F(x) par rapport à la variable aléatoire x
<F(x)>x = ∫D F(x') dP(x') où D est le domaine de définition de x et où dP(x') = probabilité de trouver la variable x entre les valeurs x' et x' +dx' .
On donne la loi de répartition en énergie de la statistique de Boltzmann :
${\rm{dP(E) = }}\frac{{\rm{1}}}{{\sqrt {{\rm{2}}\pi } }}\frac{{\rm{1}}}{{\sqrt {{{{\rm{(kT)}}}^{\rm{3}}}} }}\sqrt {\rm{E}} \,\exp \left( {E/kT} \right)\,dE$.

PREMIERE PARTIE

Dans toute cette partie,$\vec v$ et E représentant la vitesse et l'énergie cinétique du neutron, et $\vec w$ la vitesse du noyau dans le référentiel du laboratoire. L'indice 1 sera réservé aux grandeurs avant le choc et l'indice 2 aux grandeurs après le choc.
On pose A = Masse du noyau / Masse du neutron.
On se place dans l'approximation non relativiste.
I. COLLISION NEUTRON-NOYAU AU REPOS
Un neutron de vitesse ${\vec v_1}$et d'énergie E1 entre en collision élastique avec un noyau atomique initialement au repos (${\vec w_1} = \vec 0$).
1). Ecrire les équations de conservation de la quantité de mouvement et de l'énergie en fonction de A et des vitesses.
2). On appelle θ l'angle défini positif, que fait la direction du noyau après 1e choc avec la direction du neutron incident θ = (${\vec v_1},{\vec w_1}$) avec 0 < θ < π
a). Montrer que θ < π/2 .
b). Exprimer le rapport E2/E1 en fonction de A et θ.
II. MODELE DES SPHERES DURES
Dans ce modèle, on représente le neutron et le noyau par deux sphères rigides de rayons respectifs R1 et R2. On définit le paramètre d'impact b comme étant la distance entre la trajectoire du neutron incident et le centre du noyau initialement au repos. On admettra que les actions de contact, lors du choc sont normales aux surfaces de contact.
1). Donner la relation entre l'angle θ défini au I.2 et le paramètre b.
2). Exprimer la probabilité dP'(b) que le paramètre d'impact soit compris entre b et b+db au cours d'une collision.
3). Montrer que <− ln(1-K cos2θ)>b = 1 + $\frac{{1 - K}}{K}$ ln(1-K) où K est une constante vérifiant K< 1 .
4). En déduire le coefficient de ralentissement γ défini par γ = <−Ln E2/E1>b .
5). a). Pour quelle valeur de A le ralentissement est-il en principe le plus efficace?
b). Application numérique. Calculer γ pour les noyaux suivants :
1H, 2H, 12C, 238U. On assimilera A au nombre de masse du noyau.
Tourner la page
III. APPLICATION AU RALENTISSEMENT DES NEUTRONS
On considère un neutron d'énergie initiale E0 = l MeV . A l'instant t = 0 s, ce neutron entame un processus de collisions élastiques en chaîne avec un milieu, que l'on supposera homogène, illimité et constitué de noyaux tous identiques.
1). Le milieu est maintenu à la température T = 300 K. calculer l'énergie moyenne d'agitation thermique des noyaux E300K en électron-volts (la démonstration de la relation utilisée n'est pas exigée). Est-il légitime de négliger le mouvement des noyaux ?
2).a). En utilisant la définition II.4). de γ exprimer l'énergie En du neutron après n collisions en fonction de E0, γ et n. On pourra, pour justifier le calcul, considérer que n est un grand nombre.
b). Calculer, pour les 4 noyaux du II.5)., le nombre de collisions nécessaires pour faire passer l'énergie du neutron de sa valeur initiale E0 à la valeur finale E300K calculée plus haut.
3). On considère maintenant que l'énergie du neutron est une fonction continue du temps E(t). On note λ, le libre parcours moyen du neutron dans le milieu.
a). Exprimer la fréquence de collision dn/dt en fonction de λ, E(t) et m.
b). En déduire l'équation différentielle vérifiée par E(t).
c). Calculer E(t).
4). On donne, dans le cas du graphite, λ =2,6 cm.
a). Calculer le temps nécessaire pour abaisser l'énergie du neutron à la valeur finale E300K.Que pensez-vous de l'influence de E0 sur le temps de ralentissement ?
b). Calculer la distance parcourue par le neutron.

DEUXIEME PARTIE

I. CALCUL DE CHAMP MAGNETIQUE
Soit (O,$\vec i,\vec j,\vec k$) un trièdre orthonormé direct. On considère 6 fils rectilignes infinis, parallèles, de direction $\vec k$. La disposition des fils est telle que leurs traces dans le plan (O, $\vec i,\vec j$) notées A1 B3 A2 B1 A3 B2 sont réparties sur les sommets d'un hexagone régulier inscrit dans un cercle de centre O et de rayon a (voir Figure 1).
Le fil A1, tel que $O{A_1} \to = a\,\vec i$, est parcouru par un courant I dans le sens de $\vec k$ et deux fils voisins sont parcourus par des courants opposés. On veut calculer le champ magnétique créé par cette distribution de courant au voisinage de O.
1).a). Soit M un point du plan (O, $\vec i,\vec j$). on pose $OM \to $$ = \vec r$. Donner rapidement en fonction de I. a, $\vec i,\vec k$ ct$\vec r$ l'expression du champ magnétique en M créé par le fil A1 seul.
Tourner la page
b). On repère M par ses coordonnées polaires (r, θ) avec θ = ($\vec i,\vec r$). On pose u = r/a et B0 = µ0I/2πa . Donner les composantes radiale B'r et orthoradiale B'θ du champ crée par A1 seul en fonction de B0, u et θ .
c). Faire un développement limité à l'ordre 2 en u de ces expressions.
2).a). Par quelle transformation passe-t-on du champ crée par le fil A1, au champ créé par le seul fil B1 diamétralement opposé à A1 ?
b). En déduire, à l'ordre 2 en u. les composantes B1r et B du champ créé par le couple (A1, B1). I1 est conseillé, pour la suite du calcul. de linéariser les expressions en sinθ et cosθ .
3).a). Par quelle transformation obtient-on les champs créés par les deux autres couples de fils (A2, B2) et (A3, B3) ?
b). Montrer que le champ total se met sous la forme
Br = − C B0 u2 sin3θ
Bθ = − C B0 u2 cos3θ où C est une constante numérique à déterminer.
4).a). Donner l'équation des lignes de champ.
b). Représenter l'allure des lignes de champ.
5).a). Calculer le module du champ que l'on notera B(r).
b). Comment sont les lignes isomodules B(r) = Constante ?
On conviendra que cette expression de B(r) est valable pour tout r < a.
II. ACTION DU CHAMP SUR UN NEUTRON
On admet qu'un neutron placé dans un champ magnétique oriente toujours son moment magnétique dans la direction du champ mais que son sens peut-étre, de façon équiprobable, parallèle ou antiparallèle au vecteur champ magnétique.
On pose Ω = ( 2CB0M/ma2 ) 1/2
1).a). Exprimer en fonction de Ω, m et r l'énergie potentielle d'interaction entre le neutron et le champ magnétique $\vec B$déterminé dans la question I.. On distinguera le cas des neutrons "parallèles" et celui des neutrons "antiparallèles" au champ.
b). Exprimer la force qui s'exerce sur ces neutrons. En déduire qu'il est possible de confiner certains neutrons dans ce champ, on précisera lesquels.
Dans toute la suite du problème on ne s'intéresse qu'aux neutrons confinés.
2). Soit le repère d'axe cartésien (O, x, y, z) engendré par le trièdre (O, $\vec i,\vec j,\vec k$) défini au I.
a). Ecrire les équations différentielles du mouvement du neutron.
b). On considère un neutron qui, à t = 0 s. a pour coordonnées (x0. 0, 0) et pour vecteur vitesse (0, u0, v0). Exprimer x(t). y(t) et z(t).
c). Représenter sa trajectoire.
3). Soit $\vec u$ la projection du vecteur vitesse sur le plan (O, x, y).
a). Montrer qu'il existe une vitesse critique uC telle que si |$\vec u$| > uC aucun neutron n'est confiné. On exprimera uC en fonction de B0, M et m.
b). Calculer uC pour B0 = 0,5 T. Calculer l'énergie cinétique critique EC correspondante en électron-volts. Justifier, par un calcul numérique, le qualificatif d' "ultra-froids" que l'on donne aux neutrons confinés.
c). On considère un faisceau de neutrons ralentis et en équilibre thermique dans un milieu de température T = 300 K. Evaluer la fraction de ces neutrons qui sont susceptibles d'être piégés dans le champ. On utilisera la
valeur EC calculée précédemment et on fera une approximation justifiée.
4). Expliquer pourquoi. en pratique, la durée de confinement sera forcément limitée.

Tourner la page
III. AMELIORATION DU CONFINEMENT
On referme le volume cylindrique de la zone de confinement sur lui-même pour former un tore de rayon moyen R et de section circulaire de rayon a. Les 6 fils rectilignes infinis du I sont donc remplacés par 6 fils circulaires coaxiaux et on définit un nouveau repère d'axe en coordonnées cylindriques (ρ, θ, z) l'axe z étant maintenant confondu avec l'axe du tore (voir Figure 2.)
1). On admet que la configuration du champ dans une section θ = constante du tore est identique à celle déjà calculée dans le cas des fils infinis.
a). Exprimer. en coordonnées cylindriques. la force qui s'exerce sur un neutron confiné en fonction de m, Ω, R, ρ et z.
b). Exprimer les composantes de l'accélération en coordonnées cylindriques.
c). En déduire les équations différentielles du mouvement.
On choisit l'origine des temps telle que dρ/dt(t=0) = 0 et l'origine des angles telle que θ(t=0) = 0 . On pose ρ(0) = ρ0, z(0) = z0, dθ/dt (t=0) = ω0 et dz/dt (t=0) = vo.
2).a). Calculer z(t).
b). Montrer que ρ2dθ/dt est une constante du mouvement. Comment aurait-on pu prévoir directement ce résultat ?
c). En déduire une équation différentielle en ρ(t) uniquement.
3). Décrire complètement le mouvement dans les deux cas particuliers suivants :
a). ω0 = 0. Donner la nature des trajectoires.
b). dθ/dt = constante. Calculer ρ en fonction de R. Ω et ω0 et représenter les trajectoires. A quelle condition sont-elles fermées ?
4). On cherche maintenant des solutions pour lesquelles ρ varie peu autour de sa valeur moyenne.
On pose ρ(t) = ρm ( 1 + ε (t) ) avec ρm = < ρ(t) >t et | ε | << 1.
a). En ne conservant que les termes d'ordre 1 en ε, établir l'équation différentielle linéaire vérifiée par ε(t) .
b). Donner l'équation qui lie ρm aux grandeurs initiales ρ0 et ω0 . On ne cherchera pas à la résoudre.
c). En déduire les expressions de ρ(t) et de dθ /dt en fonction de ρm, ω0, Ω ,R et t.
d). Représenter l'allure des trajectoires.
5). Si on suppose l'axe z vertical, quel serait l'effet de la pesanteur sur la trajectoire du neutron ?
Faire l'application numérique pour g = 9,81 m.s−2, B0 = 0.5T et a =10 cm.
**** FIN ****

Autres Concours

2011  : Concours ENAC de  physique 2011  :  énoncé ,  corrigé Concours ICNA de  physique 2011  :  énoncé ,  corrigé Concours ICNA de ...